Đến nội dung

toanND nội dung

Có 50 mục bởi toanND (Tìm giới hạn từ 26-05-2020)



Sắp theo                Sắp xếp  

#723048 bất đẳng thức

Đã gửi bởi toanND on 14-06-2019 - 16:37 trong Bất đẳng thức và cực trị

Ngưỡng mộ anh toanND quá, anh chỉ cho em cách học giỏi bđt với ạ

Kiếm sách với tài liệu mà đọc thôi e :ukliam2:




#723098 bất đẳng thức

Đã gửi bởi toanND on 16-06-2019 - 22:23 trong Bất đẳng thức và cực trị

BÀI 3. Áp dụng BĐT Bunhiacopxki ta có $(a^2+b)(1+\frac{1}{b})\geq(a+1)^2$

Tương tự với các BĐT còn lại rồi nhân lại ta có $(a^2+b)(b^2+c)(c^2+a)\frac{(a+1)(b+1)(c+1)}{abc}\geq(a+1)^2(b+1)^2(c+1)^2$

$\Leftrightarrow (a^2+b)(b^2+c)(c^2+a)\geq abc(a+1)(b+1)(c+1)$

Dấu = xảy ra khi a =b =c 




#723109 bất đẳng thức

Đã gửi bởi toanND on 17-06-2019 - 12:51 trong Bất đẳng thức và cực trị

Áp dụng BĐT $b^2+bc+c^2\geq\frac{3}{4}(b+c)^2$

Ta có $15\geq3a^2+4(b^2+bc+c^2)\geq3[a^2+(b+c)^2]\geq\frac{3}{2}(a+b+c)^2$

$\Rightarrow (a+b+c)^2\leq10\Rightarrow-\sqrt{10}\leq a+b+c\leq \sqrt{10}$

Từ đó ta có min , max

e tự tìm dấu = nhé




#723037 bất đẳng thức

Đã gửi bởi toanND on 14-06-2019 - 13:22 trong Bất đẳng thức và cực trị

Em cảm ơn anh toannd, bài 1 câu a anh có thể giải bằng cauchy dc k ah

Có thể dùng AM - GM (Cauchy) kiểu này : $\frac{1}{a}+\frac{1}{b}\geq\frac{4}{a+b}; \frac{1}{ab}\geq\frac{4}{(a+b)^2}$

Áp dụng hai BĐT trên, ta biến đổi biểu thức P như sau:

$P=(1+\frac{1}{a}+\frac{1}{b}+\frac{1}{ab})(1+\frac{1}{c}+\frac{1}{d}+\frac{1}{cd})\geq[1+\frac{4}{a+b}+\frac{4}{(a+b)^2}][1+\frac{4}{c+d}+\frac{4}{(c+d)^2}]=[(\frac{2}{a+b}+1)(\frac{2}{c+d}+1)] ^2 ]$

Đặt $A=[(\frac{2}{a+b}+1)(\frac{2}{c+d}+1)]^2$

$\Rightarrow A=[\frac{4}{(a+b)(c+d)}+\frac{2}{a+b}+\frac{2}{c+d}+1]^2\geq[\frac{16}{(a+b+c+d)^2}+\frac{8}{a+b+c+d}+1]^2=625$

$\Rightarrow P\geq A\geq625$  ~O)




#723026 bất đẳng thức

Đã gửi bởi toanND on 14-06-2019 - 08:50 trong Bất đẳng thức và cực trị

BÀI 1

a. Áp dụng BĐT Holder ta có: $P=(\frac{1}{a}+1)(\frac{1}{b}+1)(\frac{1}{c}+1)(\frac{1}{d}+1)\geq (\sqrt[4]{\frac{1}{abcd}}+1)^{4}$

Mặt khác theo BĐT AM-GM: $\sqrt[4]{abcd}\leq \frac{a+b+c+d}{4}=\frac{1}{4}$

$\Rightarrow P\geq(4+1)^{4}=625$

Vậy $minP=625$ khi $a=b=c=d= \frac{1}{4}$

b. Ta có $Q=\frac{a(b+c+d)}{\frac{a^{2}}{3}+b^{2}+\frac{a^{2}}{3}+c^{2}+\frac{a^{2}}{3}+d^{2}}\leq\frac{a(b+c+d)}{\frac{2}{\sqrt{3}}(ab+ac+ad)}=\frac{\sqrt{3}}{2}$

Vậy $maxQ = \frac{\sqrt{3}}{2}$ khi ..........

BÀI 2. Ý tưởng cũng giống bài 1b thôi e  :closedeyes:




#723698 Một số bài toàn bất đẳng thức mình cần được giúp đỡ!

Đã gửi bởi toanND on 12-07-2019 - 16:52 trong Bất đẳng thức và cực trị

$\boxed{16}$

Áp dụng BĐT Cauchy Schwarz ta có

$\frac{a}{4b^2+1}+\frac{b}{4c^2+1}+\frac{c}{4a^2+1}=\frac{a^3}{4a^2b^2+a^2}+\frac{b^3}{4b^2c^2+c^2}+\frac{c^3}{4c^2a^2+c^2}\geq \frac{(a\sqrt{a}+b\sqrt{b}+c\sqrt{c})^2}{4(a^2b^2+b^2c^2+c^2a^2)+a^2+b^2+c^2}$

Ta chỉ cần chứng minh 

$4(a^2b^2+b^2c^2+c^2a^2)+a^2+b^2+c^2\leq 1 =(a+b+c)^2$

$\Leftrightarrow ab(1-2ab)+bc(1-2bc)+ca(1-2ca)\geq0$

Lại có $ab>0$ , $ab\leq\frac{(a+b)^2}{4}<\frac{(a+b+c)^2}{4}$ $\Rightarrow ab<\frac{1}{4}\Rightarrow ab(1-2ab)>0$

Tương tự ta có đpcm

Dấu = không xảy ra




#722862 BDT

Đã gửi bởi toanND on 08-06-2019 - 16:49 trong Bất đẳng thức và cực trị

mình nghĩ là có đk a,b,c không âm




#723404 Hê thức lượng

Đã gửi bởi toanND on 29-06-2019 - 22:28 trong Hình học

$\boxed{3}$

Kẻ EK, FI lần lượt vuông góc với AB, AC tại K, I.

Ta có $\frac{AK}{BK}=\frac{AK.AB}{BK.AB}=\frac{EA^2}{EB^2}=4=\frac{HC}{HB}$

theo định lý Thales đảo $\Rightarrow HK||AC$. Tương tự $HI\parallel AB$

$\Rightarrow AIHK$ là hình bình hành

Ta có $\bigtriangleup AEB\sim \bigtriangleup CFA \Rightarrow \measuredangle BAE=\measuredangle ACF=\measuredangle AFI\Rightarrow \bigtriangleup AEK\sim \bigtriangleup FAI$

$\Rightarrow \frac{EK}{AK}=\frac{AI}{FI}\Rightarrow \frac{EK}{AI}=\frac{AK}{FI}$. Mà $AK = HI,AI=HK$

$\Rightarrow \frac{EK}{HK}=\frac{HI}{IF}$. Lại có $\measuredangle EKH=\measuredangle BKH +90^0=\measuredangle HIC+90^0=\measuredangle HIF$

Do đó $\bigtriangleup EKH\sim\bigtriangleup HIF\Rightarrow\measuredangle HEK=\measuredangle IHF$

Vậy $\measuredangle EHF=\measuredangle EHK+\measuredangle KHI+\measuredangle IHF=(\measuredangle EHK+\measuredangle HEK)+\measuredangle BKH=180^0-\measuredangle EKH+\measuredangle BKH=180^0-90^0=90^0$

 




#723413 Hê thức lượng

Đã gửi bởi toanND on 30-06-2019 - 16:14 trong Hình học

Em cảm ơn anh toanND, em check lại bài 2 đề bài vẫn đúng ah

Ở câu b/ nếu $\angle BCA=90^0$ thì tam giác ABC có hai góc vuông à?




#723405 Hê thức lượng

Đã gửi bởi toanND on 29-06-2019 - 22:33 trong Hình học

e coi lại đề bài 2 xem 

nhimtom



#723400 Hê thức lượng

Đã gửi bởi toanND on 29-06-2019 - 17:40 trong Hình học

$\boxed{\text{1}}$

hethuc1.PNG

a. Dễ thấy tứ giác KIHB nội tiếp $\rightarrow CI.CK=CH.CB=CA^2$

b. theo tính chất đối xứng thì $\measuredangle BDC=\measuredangle BAC=90^0;AC=DC$

Vì $\measuredangle BKC=\measuredangle BAC=\measuredangle BDC=90^0$ nên $B,K,A,C,D$ cùng thuộc một đường tròn

$\Rightarrow \measuredangle CKD=\measuredangle DAC=\measuredangle ADC=\measuredangle AKC$

ta có đpcm

 




#723414 Hê thức lượng

Đã gửi bởi toanND on 30-06-2019 - 16:49 trong Hình học

$\boxed{4}$

hethuc2.PNG

a. Ta có $BK^2=BA^2=BH.BC\Rightarrow \bigtriangleup BKH \sim \bigtriangleup BCK\Rightarrow \widehat{BKH}=\widehat{BCK}$

   Tương tự $\bigtriangleup DKL \sim \bigtriangleup DCK\Rightarrow \widehat{DKL}=\widehat{BCK}$

   Vậy $\widehat{BKH}=\widehat{DKL}$

b. Dựng hai đường tròn $(B,BK)$  $(D,DK)$. Gọi T là giao điểm thứ hai của hai đường tròn này.

    Gọi M, N lần lượt là giao điểm của TK với AC, BC.

    Dễ thấy AC là tiếp tuyến chung của (B,BK) và (D, DK)

    Ta có $MA^2=MK.MT=MI^2\Rightarrow MA=MI$. Lại có $MN\parallel AH\parallel IL$ (cùng vuông BC) nên NH = NL.

    Mặt khác $KN\perp HL$ nên tam giác HKL cân $\Rightarrow KH=KL$

c. Từ $\bigtriangleup DKL \sim\bigtriangleup DCK$ kết hợp với KH = KL ta có $\widehat{KHC}=\widehat{KLH}=\widehat{DKC}$

    Do đó $\bigtriangleup CKD\sim\bigtriangleup CHK\Rightarrow CK^2=CD.CH$. Lại có tứ giác AIDH nội tiếp nên $CD.CH=CI.CA$

    Vậy $CK^2=CA.CI$  ~O)

 




#722861 chứng minh bất đẳng thức

Đã gửi bởi toanND on 08-06-2019 - 16:22 trong Bất đẳng thức và cực trị

Ta đi chứng minh BĐT sau : $\frac{a}{a^{2}+1}\leq \frac{18}{25}a+\frac{3}{50}$ với $a\geq 0$.

Sau khi thu gọn thì bất đẳng thức trên tương đương với $(3a-1)^{2}(4a+3)\geq 0$ luôn đúng với mọi a không âm.

Làm tương tự với các BĐT còn lại rồi cộng lại ta được đpcm. 

Dấu đẳng thức xảy ra khi $a=b=c=\frac{1}{3}$




#722873 chứng minh bất đẳng thức

Đã gửi bởi toanND on 09-06-2019 - 08:25 trong Bất đẳng thức và cực trị

Cách a cũng giống 

Sin99



#722942 Chứng minh rằng B, E, R thẳng hàng.

Đã gửi bởi toanND on 10-06-2019 - 21:25 trong Hình học

Anh thấy có nhiều cách giải mà, đâu chỉ mỗi Pascal đâu !!

attachicon.gifScreenshot from 2019-06-10 21-18-36.png

Tại em thấy bài này trong bài tập về định lý Pascal nên mới tìm cách giải bằng Pascal nhưng lại chưa ra. Nhưng cũng cảm ơn a.




#722933 Chứng minh rằng B, E, R thẳng hàng.

Đã gửi bởi toanND on 10-06-2019 - 17:25 trong Hình học

$\boxed{\text{APMO 2013}}$ Cho tứ giác ABCD nội tiếp đường tròn (O). P nằm trên tia AC sao cho PB, PD tiếp xúc với (O). Tiếp tuyến của (O) tại C cắt PD tại Q, AD tại R. E là giao điểm thứ hai của AQ và (O). Chứng minh rằng B, E, R thẳng hàng.

Hình gửi kèm 

APMO 2013.PNG




#722939 Chứng minh rằng B, E, R thẳng hàng.

Đã gửi bởi toanND on 10-06-2019 - 21:16 trong Hình học

Có cách nào sử dụng định lý Pascal không nhỉ?




#724550 CMR $ TM // BC $

Đã gửi bởi toanND on 08-08-2019 - 09:38 trong Hình học

Bài này có thể giải bằng định lý Pascal




#722758 Hướng vẽ hình bằng Geogebra

Đã gửi bởi toanND on 05-06-2019 - 10:11 trong Thử các chức năng của diễn đàn

[attachment=Capture 6.png]




#724419 $1+\sqrt{\frac{2+1}{2}}+\sq...

Đã gửi bởi toanND on 01-08-2019 - 21:05 trong Bất đẳng thức và cực trị

 chỗ $\left ( 1+\frac{1}{n^{2}} \right )^{n}> 1+n.\frac{1}{n^{2}}$ này là sao bạn

Bất đẳng thức Bernoulli đó




#722765 Chứng minh BK chia đôi MF.

Đã gửi bởi toanND on 05-06-2019 - 11:11 trong Hình học

Capture 7.PNG

dễ thấy E là điểm chính giữa cung lớn BC nên E, O, M thẳng hàng và $EM\perp BC$.

Gọi I là giao điểm của FM với AC. Dễ thấy FM là đường thẳng Simson của tam giác ABC nên $EI \perp AC$.

Lại có $\widehat{FAE}=\widehat{FEI}\Rightarrow \Delta AEF=\Delta AEI\Rightarrow AF=AI,EF=EI$

do đó AE là trung trực của FI $\Rightarrow AK\perp FM$. (1)

MN là đường trung bình của tam giác ABC$\Rightarrow MN||AB\Rightarrow MK\perp EF$ (2)

Từ (1) và (2) suy ra E là trực tâm của tam giác MFK $\Rightarrow EM\perp FK\Rightarrow FK||BM$

Suy ra FKMB là hình bình hành $\Rightarrow đpcm$




#722132 Chứng minh rằng số đo góc MPN luôn không đổi khi D thay đổi

Đã gửi bởi toanND on 11-05-2019 - 12:27 trong Hình học

Cho tam giác ABC, giả sử có điểm P nằm trong tam giác ABC sao cho góc BPC = góc CPA = góc APB. PB, PC theo thứ tự cắt CA, AB tại E, F. D là điểm di chuyển trên cạnh BC. Đường thẳng DF cắt đường thẳng AC tại M. Đường thẳng DE cắt đường thẳng AB tại N.

1. Chứng minh rằng số đo góc MPN luôn không đổi khi D thay đổi.

2. Gọi giao điểm của đường thẳng EF với đường thẳng MN là Q. Chứng minh rằng PQ là phân giác của góc MPN.

Capture 4.PNG




#722159 Chứng minh rằng số đo góc MPN luôn không đổi khi D thay đổi

Đã gửi bởi toanND on 12-05-2019 - 17:01 trong Hình học

Xét phép nghịch đảo tâm $P$ phương tích bất kì, ta đưa bài toán đã cho về bài toán mới sau.

Bài toán mới. Cho $\Delta ABC,P$ là điểm trong tam giác thoả $\widehat{APB}= \widehat{BPC}= \widehat{CPA}.PB,PC$ theo thứ tự cắt $(CPA),(APB)$ tại $E,F.D$ là điểm di chuyển trên $(PBC),(PDF)$ cắt $(PAC)$ tại $M,(PDE)$ cắt $(PAB)$ tại $N.$

a) Chứng minh số đo góc $\widehat{MPN}$ không đổi.

b) $(PEF)$ cắt $(PMN)$ tại $Q.$ Chứng minh $PQ$ là phân giác $\widehat{MPN}.$

Chứng minh.

a) Gọi $G,H,I,J,K,L,O$ là tâm các đường tròn $(PAC),(PAB),(PDF),(PDE),(PBC),(PEF),(PMN).$

Từ cách xác định các điểm này, ta có các bộ điểm thẳng hàng: $\overline{L,H,I}, \overline{I,K,J}, \overline{J,G,L}, \overline{I,O,G}, \overline{J,O,H}.$

Ta cũng có $IL \perp PC,JL \perp PB,GK \perp PC,HK \perp PB,HG \perp PA.$ Với chú ý $\widehat{APB}= \widehat{BPC}= \widehat{CPA}=120^0,$ ta được $\Delta KGH$ đều và $\widehat{KHI}= \widehat{KGI}=60^0.$

Do đó $IH \parallel GK,HK \parallel GJ \Rightarrow \Delta HIK \sim \Delta GKJ \Rightarrow \frac{HI}{HG}= \frac{HI}{HK}= \frac{GK}{GJ}= \frac{GH}{GJ} \Rightarrow \Delta IHG \sim \Delta HGJ$

$\Rightarrow \widehat{IOH}= \widehat{OGH}+ \widehat{OHG}= \widehat{OHG}+ \widehat{OJG}= 180^0- \widehat{HGJ}=60^0.$

Mà $MP \perp OI,NP \perp OH \Rightarrow \widehat{MPN}=60^0.$ Ta có đpcm.

b) Theo cách xác định các tâm đường tròn ở câu a), $\Delta HLG$ đều. Lại có $\widehat{IOH}=60^0$ theo câu a) nên $L$ là trung điểm cung $HG$ không chứa $O$ của $(HOG) \Rightarrow OL$ là phân giác $\widehat{HOG}.$

Lại có $MP \perp OG,NP \perp OH,PQ \perp OL \Rightarrow PQ$ là phân giác $\widehat{MPN}.$ Ta có đpcm.

 

có cách nào k sử dụng phép nghịch đảo k ạ? Tại e chưa học cái này




#721264 $\sum \frac{a+b}{c} \geq 2\sqrt{( \sum a)(...

Đã gửi bởi toanND on 03-04-2019 - 23:52 trong Bất đẳng thức - Cực trị

Cho các số thực dương a,b,c. Chứng minh rằng khi đó ta có:

$\frac{a+b}{c}+\frac{b+c}{a}+\frac{c+a}{b}\geqslant 2\sqrt{(a+b+c)(\frac{a}{bc}+\frac{b}{ca}+\frac{c}{ab})}$




#722771 Dựng đường tròn $\omega _{1}$

Đã gửi bởi toanND on 05-06-2019 - 17:22 trong Hình học

 

b) Cách dựng $\omega_{3}:$ Gọi tâm nội tiếp $\Delta DAE$ là $L,O_3$ là hình chiếu $L$ lên phân giác trong $\widehat{ACD}.$ Khi đó đường tròn tâm $O_3$ tiếp xúc $CA$ chính là $\omega_{3}$ (bổ đề Sawayama-Thebault).

sao k đúng nhỉ?

@halloffame: Mình chỉnh lại rồi nhé.